This page shows a recording of a live class. We're working hard to create our standard, concise explanation videos for the questions in this PrepTest. Thank you for your patience!

1 comment

This page shows a recording of a live class. We're working hard to create our standard, concise explanation videos for the questions in this PrepTest. Thank you for your patience!

This is a method of reasoning “except” question, which means there will be 4 answer choices that are method of reasoning answer choices that are correct and one that is not a method of reasoning used in the argument - the latter is the correct answer. We know this is a MR question because of the questions stem: In order to advance her point of view, the author does all of the following EXCEPT...

The first sentence gives us a question: do people who pluck their gray hairs out have a more negative attitude toward older people than those who don’t care about their grays? The answer is given in a conditional: If a person’s psychopathology does not make them overgeneralize, there is no connection. This is because it’s fine to dislike the idea of getting older while liking elderly people. The other reason is that it’s immoral to dislike older people just because they’re old, according to the stimulus. Adding onto this, there is nothing wrong about disliking the idea of getting older. That conditional is our conclusion and everything after that is support given for this conclusion.

Answer Choice (A) describes what the conclusion is doing - it’s saying the assertion posed in the question is not necessary.

Answer Choice (B) is what the author is doing in the third sentence: “Clearly, it is reasonable...”

Answer Choice (C) is a method of reasoning employed in the stimulus - the general principle is the fact that it is immoral to be ageist.

Correct Answer Choice (D) is a method of reasoning that is NOT used in the stimulus. We’re leaning into stereotypes of the elderly (graying, eyesight), not discrediting them.

Answer Choice (E) is a method used - it’s the fourth sentence.


Comment on this

This page shows a recording of a live class. We're working hard to create our standard, concise explanation videos for the questions in this PrepTest. Thank you for your patience!

This is a weakening question, though I’d forgive you if you failed to immediately identify it as it throws a particularly dense question stem at us. This is one of those question stems that introduces a final premise which is to be added to the stimulus in our judgment of the answer choices, but in this case, it does so in an incredibly confusing way. Our objective is to specifically weaken an interpretation that holds that the lines serve unrelated purposes rather than refer to astronomical phenomena. Despite the density of this question stem, what should tip us off that it is specifically a weakening question is that it asks: which one of the following, if true, most effectively counters.

The first sentence of the stimulus is very strangely worded, but what we should pick up on is that these lines are ancient, about as wide as a footpath, and stretch long distances; sounds like some old roads! We next learn that these lines form giant shapes, and that one where a bunch of lines emerge from a point intersects with one shaped like a bird; ok not sounding so much like roads anymore. The rest of the stimulus concerns the interpretation of an investigator who thinks the markings are landing strips for aliens and argues that they couldn’t have been roads. From him, we learn a bunch of information that casts doubt on the roads hypothesis; the lines run in strange patterns and will sometimes just end in the middle of the desert. Our job is to weaken an explanation of these weird lines which interprets their shapes as serving unrelated purposes, and specifically to do so from a perspective that believes the lines have an astronomical purpose. Let’s see what we get in the answer choices:

Answer Choice (A) Interesting, but we are trying to weaken the interpretation that these specific lines in Peru had no purpose; what North American peoples do isn’t of interest to us.

Correct Answer Choice (B) This answer strongly suggests that the lines have an astronomical purpose which unites the straight lines (observation spot) with the figure (represents a constellation).

Answer Choice (C) This fails to weaken the interpretation that the figures are unrelated; in fact, it seems to suggest the straight lines are part of a complex of patterns and have nothing to do with the bird.

Answer Choice (D) Similar to A, this brings in a different phenomenon and does nothing to explain how the straight lines and bird figure could have together served an astronomical purpose.

Answer Choice (E) This strengthens the hypothesis we want to weaken; if one was made well before the other, it’s far more likely they were unrelated.


Comment on this

This is an EXCEPT must be true question, since the stem asks: If the statements above are true, each of the following must also be true on the basis of them EXCEPT:

Our stimulus begins with a some statement, and informs us that some of the world’s most beautiful cats are Persian cats. It’s important to remember that just because some beautiful cats are Persian, that doesn’t mean all Persian cats are beautiful. What we do learn about all Persian cats however, is that they are pompous. I’m not quite sure what it means for a cat to be pompous (I’m picturing a cat turning up its nose at dry food), but we also learn that pompous cats are invariably irritating, which means that pompous cats are always irritating. Because this is an EXCEPT must be true question, we know we are going to get four answers that are guaranteed to be true from this cat info, and that the correct answer will be the one answer which could be false. Let’s see what cat inferences our answer choices make:

Answer Choice (A) If all Persian cats are irritating, and some of the most beautiful cats are Persian, then it must be true that some of the most beautiful cats are irritating.

Answer Choice (B) Same as A, this is guaranteed

Answer Choice (C) All Persian cats are Irritating means that Persian → Irritating. C is just the contrapositive of this conditional, and therefore must be true.

Answer Choice (D) If all Persian cats are pompous, and some of the most beautiful cats are Persian, then it must be true that some of the most beautiful cats are pompous.
Correct Answer Choice (E) This could be false! For all we know all cats that are both irritating and beautiful are Persian.


Comment on this

This page shows a recording of a live class. We're working hard to create our standard, concise explanation videos for the questions in this PrepTest. Thank you for your patience!

We should recognize this as a weakening question: Which one of the following, if true, most seriously weakens the representative’s argument?

Our stimulus is attributed to a Beverage company representative, and he makes a prediction about the effect of a new policy. Thus far, beverage cans have been linked by plastic rings that often harm and suffocate animals when they end up in the environment. All the beverage companies, following the representative’s company’s lead, are going to adopt new plastic rings that disintegrate after three days exposure to sunlight. Sounds like a much better option! The representative concludes with a prediction that once the switchover from the old to the new plastic rings is completed, the threat of suffocation to animals will be eliminated. Our job is to weaken this prediction: we want an answer choice that would reduce the probability that the change, once adopted, will be sufficient to eliminate the threat to animals. On to the answers:

Answer Choice (A) Remember, our representative’s prediction specifies that the elimination will occur once the switchover is complete.

Correct Answer Choice (B) If the original problem is still out there, well then even if it is no longer going to be getting worse a switchover won’t be enough to eliminate the threat of plastic rings to animals.

Answer Choice (C) This doesn’t weaken the prediction at all.

Answer Choice (D) Good for the new rings, but this does nothing to weaken the prediction.

Answer Choice (E) Okay this gives us a reason why the change will harm animals, but remember, we are specifically interested in whether it will eliminate the problem of suffocating.


Comment on this

This page shows a recording of a live class. We're working hard to create our standard, concise explanation videos for the questions in this PrepTest. Thank you for your patience!

This is a strengthening question, though it may be difficult to identify. We should know it is a strengthening question because the stem asks: On the basis of the premises advanced, which one of the following principles, if established, would provide the most justification for the concluding recommendation?

The stimulus begins with the statistic that 99% of the burglar alarms police respond to are false alarms. Seems like a big waste of time! The stimulus explicitly tells us that this is a drain on important resources, with each false alarm wasting an average of ~45 minutes of police time. This draws police attention away from other crimes and mostly benefits businesses and the rich, but on the other hand alarm systems are effective deterrents of burglaries. The author concludes by recommending a compromise: police keep responding to burglar alarms but fine alarm owners the cost for false alarms. Our job is to find the principle in the answer choices will justify the author’s recommendation. Let’s see our options:

Answer Choice (A) This principle would justify eliminating burglar alarm systems, not the author’s compromise.

Answer Choice (B) The author is recommending that burglar systems owners be fined in specifically the case of false alarms, regardless of whether police resources are in short supply or they can afford to.

Answer Choice (C) We haven’t been told anything about whether the author’s recommendation would reduce crime levels throughout the entire area the police serve, just that they are deterrent for their owners!

Answer Choice (D) Important to remember the condition for people being required to pay is if they waste police time with a false alarm, not whether they directly benefit from the police service

Correct Answer Choice (E) We’ve been explicitly told that false alarms waste police time which could be spent on other legitimate calls, so this would justify the author’s recommendation that false alarm owners be fined.


Comment on this

This page shows a recording of a live class. We're working hard to create our standard, concise explanation videos for the questions in this PrepTest. Thank you for your patience!

We should recognize this as a must be true question, as it asks: If both G’s assertions and H’s assertions are true, which one of the following must also be true*?*

This MBT question takes the form of a dialogue between “G” and “H”. G thinks the Met art show is biased because more photographs were shown despite equal submissions between photographs, sculptures, and paintings. So it seems like photographs were selected at higher rates despite applying at the same rate. H counters this accusation of bias by informing us that all and only those works that met the traditional criteria were submitted. An all and only statement indicates a biconditional, in this case: met criteria ←→ exhibited. Every application that met the criteria was exhibited, and every exhibited artwork met the criteria. In that case it seems like, rather than bias, more sculptures and paintings just didn’t meet the traditional criteria this year. Since the criteria is traditional, we can infer it is the same as prior years, and therefore doesn’t represent a change that could be biased. Let’s see the answer choices:

Answer Choice (A) This must be false as we were told there were equal numbers of submissions between categories, and each artist was only allowed to submit work in one category.

Answer Choice (B) All the exhibited artworks did, but since all the artworks that met the criteria were exhibitted, and not all submitted works were exhibited, then some submitted works did not meet the traditional criteria.
Answer Choice (C) We are told nothing about comparative quality.

Answer Choice (D) For all we know all submitted photographs were accepted.

Correct Answer Choice (E) Bingo! Based on our biconditional and the fact that more photographs were exhibited, more photograph submissions must have met the traditional criteria.


1 comment

This page shows a recording of a live class. We're working hard to create our standard, concise explanation videos for the questions in this PrepTest. Thank you for your patience!

This is a must be true question, as it asks: If the statements above are all true, which one of the following must also be true on the basis of them?

We learn that Jennifer, an employee of three years, is entitled to four weeks of paid vacation and plans to spend all of it on vacation with her family. Good for Jennifer! The next sentence tells us that anyone who has worked between one and four years is automatically entitled to three weeks paid vacation a year but can apply up to half of any vacation time leftover in a year to the next year’s vacation. This must apply to Jennifer, so three weeks of her four weeks vacation time must be because she has worked between one and four years. An important word in this stimulus is exactly. From this we can infer that this three weeks is all the entitled vacation time workers get excepting from applying leftover time, and therefore if Jennifer has four weeks, it must because she had leftover vacation time from last year. And that’s all the information we get! On to the answers:

Correct Answer Choice (A) This is exactly what we identified in our stimulus reading. If she has more than three weeks, it must be from applying half of last’s years unsaved time, which must have been two weeks.

Answer Choice (B) Next year Jennifer will be beginning her fifth year, and we are only told about the vacation time of employees of between one and four years.

Answer Choice (C) We haven’t been told anything about what employees other than Jennifer are doing.

Answer Choice (D) Then we would expect her to have four and a half weeks this year

Answer Choice (E) We have no justification to assume this must be true.


2 comments

This page shows a recording of a live class. We're working hard to create our standard, concise explanation videos for the questions in this PrepTest. Thank you for your patience!

This is a weakening question, as the question stem asks: Which one of the following, if true, casts the most doubt on the accuracy of the above conclusion?

This stimulus is placed in quotes without an identified speaker, but this doesn’t really matter beyond allowing some of the referential phrasing such as this company. It begins with the context that the speaker’s company will not be training more pilots, as they have a waitlist of 400 trained pilots. Alright, that kind of makes sense. This decision is then supported by an argument.

The argument begins with the premise that the five other major companies have roughly ~ 400 pilots as well. The stimulus continues with another premise with the support indicator since, stating that each company is going to only need roughly ~100 pilots. From these two premises about the company waitlists and personnel needs, the author concludes that there will be no shortage of personnel. The assumption this argument depends on is that the individuals waitlisted for each company are distinct. Since each company requires about ~100 people and there are 5 companies, around ~500 pilots are needed by the major companies. For the argument to conclude that there are enough trained pilots to avoid a shortage, it must assume that it isn’t the same 400 pilots waitlisted for each company, as in that case there would actually already be a shortage brewing since there are ~100 less pilots than needed. An answer which picks up on this assumption and contradicts it will be an excellent weakening answer. Let’s see what we get:

Correct Answer Choice (A) This does exactly what we identified with our pre-phrase, it directly contradicts the arguments assumption that there isn’t significant overlap between the waitlists.

Answer Choice (B) Our author’s argument is specifically about the forseeable future, the argument’s conclusion is not about whether or not training will be needed in the long run.

Answer Choice (C) Ok? Our author’s conclusion is a prediction that there will be no shortage in the foreseeable future; whether or not there will be an age gap does nothing to weaken the argument.

Answer Choice (D) If anything this strengthens the argument by supporting the accuracy of the author’s claims.

Answer Choice (E) If other companies are training more pilots, that strengthens the author’s prediction that there won’t be a shortage.


1 comment

This is a resolve reconcile explain question, though it may be difficult to identify. The phenomenon we are trying to explain is how the statistic could be accurate if the conclusion isn’t, i.e. how could emergency room visits grow if heroin use remained constant or declined. The question stem asks us: Which one of the following, if true, would account for the statistic above without supporting the author’s conclusion?

The statistic mentioned in the stem is that the amount of emergency room visits by heroin users increased by 20% during the 80s. The author concludes that this was a result of an increase in the use of heroin, but we want to explain why the phenomenon identified by the statistic occurred without supporting this conclusion, so our hypothesis can’t involve an increase in heroin use. We are therefore looking for an alternative hypothesis. Let’s see what we get in the answer choices.

Correct Answer Choice (A) Bingo! This answer gives us a reason for why more heroin users would end up in the hospital even if there wasn’t an increase in heroin use, namely, increased violence from the drug trade.

Answer Choice (B) If it reduced the risk of infection, we would expect a decrease in the number of hospital visits.

Answer Choice (C) Interesting, but this does nothing to explain why there was such an increase in the first place.

Answer Choice (D) Remember, we don’t want to support the conclusion that heroin use increased!

Answer Choice (E) This gives us a more detailed explanation of what specific issues caused heroin users to go to the emergency room, but not why the amount of them who did so increased. This is a more complete description of the phenomenon, not a hypothesis for why it occurred.


Comment on this